OPTADS360
AANETWORK
AMBIENT
YOMEDIA
Banner-Video
IN_IMAGE
  • Câu hỏi:

    Trong không gian Oxyz, cho mặt cầu (S) tâm I (9; 3; 1) bán kính bằng 3. Gọi M, N là bài điểm lần lượt thuộc hai trục Ox, Oz sao cho đường thẳng MN tiếp xúc với (S), đồng thời mặt cầu ngoại tiếp tứ diện OIMN có bán kinh bằng \(\frac{13}{2}\). Gọi A là tiếp điểm của MN và (S), giá trị AM.AN bằng

    • A. 

      \(12\sqrt{3}\).

    • B. 
      18. 
    • C. 
      \(28\sqrt{3}\).
    • D. 
      39. 

    Lời giải tham khảo:

    Đáp án đúng: A

    I(9;3;1) => d(i(Oxz)) = 3 = R =? (S) tiếp xác với (Oxz).

    Gọi M (a; 0 ;0) \(\in\) Ox

    N (0; 0; b) \(\in\) Oz

    MN tiếp xác với (S) tại A nên A là hình chiếu của I lên (Oxz)

    Suy ra A (9; 0; 1)

    Gọi K là trung điểm MN => K (a/2; 0; b/2)

    Gọi H là tâm mặt cầu ngoại tiếp tứ diện OIMN => OH = 13/2 => HK \(\bot\) MN

    Gọi T là trung điểm OM => \(\left\{ \begin{array}{l}
    OM \bot KT\\
    OM \bot HT
    \end{array} \right. \Rightarrow OM \bot (KHT) =  > OM \bot HK =  > HK \bot (OMN)\)

    Mà IA \(\bot\) (OMN) => HK // IA

    Ta có \(\overrightarrow {AI}\) = (0;3;0)

    \(\overrightarrow {KH}  = \left( {{x_H} - \frac{a}{2};{y_H} - 0;{z_H} - \frac{b}{2}} \right)\)

    \(\overrightarrow {AI}\) cùng phương \(\overrightarrow {KH}\) nên \(\left\{ \begin{array}{l}
    {x_H} - \frac{a}{2}\\
    {y_H} - c\\
    {z_H} - \frac{b}{2}
    \end{array} \right.\left( {c \ne 0} \right)\)

    => \(H\left( {\frac{a}{2};c;\frac{b}{2}} \right)\)

    \(\begin{array}{l}
    OH = \frac{{13}}{2} =  > \frac{{{a^2}}}{4} + {c^2} + \frac{{{b^2}}}{4} = \frac{{169}}{4}(1)\\
    HI = OH = \frac{{13}}{2} =  > {\left( {\frac{a}{2} - 9} \right)^2} + {\left( {c - 3} \right)^2} + {\left( {\frac{b}{2} - 1} \right)^2} = \frac{{169}}{4}(2)
    \end{array}\)

    Từ (1) và (2) suy ra \(\frac{{{a^2}}}{4} + {c^2} + \frac{{{b^2}}}{4} = {\left( {\frac{a}{2} - 9} \right)^2} + {\left( {c - 3} \right)^2} + {\left( {\frac{b}{2} - 1} \right)^2}\)

    => 9a + b + 6c = 91 (3)

    \(\begin{array}{l}
    \overrightarrow {AM}  = (a - 9;0; - 1)\\
    \overrightarrow {AN}  = ( - 9;0;b - 1)
    \end{array}\)

    A, M, N thẳng hàng \(\frac{{a - 9}}{{ - 9}} = \frac{{ - 1}}{{b - 1}}\)

    ⇔ (a-2)(b-1) = 9

    ⇔ ab - a - 9b + 9 = 9

    ⇔ ab -a - 9b = 0

    ⇔ a(b-1) = ab

    ⇔ \(a = \frac{{9b}}{{b - 1}}\)

    Từ (3) suy ra

    \(\begin{array}{l}
    9.\frac{{9b}}{{b - 1}} + b + 6c = 91\\
    \frac{{81b}}{{b - 1}} + b + 6c = 91\\
     \Leftrightarrow \frac{{{b^2} + 80b}}{{b - 1}} + 6c = 91 \Leftrightarrow 91 - \frac{{{b^2} + 80b}}{{b - 1}} = \frac{{ - {b^2} + 11b - 91}}{{b - 1}}\\
     \Leftrightarrow c = \frac{{ - {b^2} + 11b - 91}}{{6\left( {b - 1} \right)}}
    \end{array}\)

    Ta có: \({a^2} + 4{c^2} + {b^2} = 169\)

    \(\begin{array}{*{20}{l}}
    { \Leftrightarrow {{\left( {\frac{{9b}}{{b - 1}}} \right)}^2} + 4\left( {\frac{{ - {b^2} + 11b - 91}}{{6\left( {b - 1} \right)}}} \right) + {b^2} = 169}\\
    { \Leftrightarrow 9.81{b^2} + \left( {{b^4} + 121{b^2} + 8281 - 22{b^3} + 182{b^2} - 2002b} \right) + 9{b^2}\left( {{b^2} - 1} \right) = 169.9.{{\left( {b - 1} \right)}^2}}\\
    { \Leftrightarrow 729{b^2} + {b^4} + 121{b^2} + 8281 - 22{b^3} + 182{b^2} - 2022b + 9{b^4} - 18{b^3} + 9{b^2} = 1521{b^2} - 3042b + 1521}\\
    { \Leftrightarrow 10{b^4} - 40{b^3} - 480{b^2} + 1040b + 6760 = 0}\\
    { \Leftrightarrow \left[ {\begin{array}{*{20}{l}}
    {b = 1 + 3\sqrt 3  =  > a = \frac{{9\left( {1 + 3\sqrt 3 } \right)}}{{ - 3\sqrt 3 }} = 9 + \sqrt 3 }\\
    {b = 1 - 3\sqrt 3  =  > a = \frac{{9\left( {1 - 3\sqrt 3 } \right)}}{{ - 3\sqrt 3 }} = 9 - \sqrt 3 }
    \end{array}} \right.}
    \end{array}\)

    TH1: 

    \(\begin{array}{l}
    a = 9 + \sqrt 3 ;b = 1 + 3\sqrt 3  =  > \overrightarrow {AM}  = \left( {\sqrt 3 ;0; - 1} \right) =  > AM = 2\\
     =  > \overrightarrow {AN}  = \left( { - 9;0;3\sqrt 3 } \right) =  > AN = \sqrt {108} \\
    AM.AN = 2.\sqrt {108}  = 12\sqrt 3 
    \end{array}\)

    TH2:

    \(\begin{array}{l}
    a = 9 - \sqrt 3 ;b = 1 - 3\sqrt 3  =  > \overrightarrow {AM}  = \left( { - \sqrt 3 ;0; - 1} \right) =  > AM = 2\\
     =  > \overrightarrow {AN}  = \left( { - 9;0; - 3\sqrt 3 } \right) =  > AN = \sqrt {108} \\
    AM.AN = 2.\sqrt {108}  = 12\sqrt 3 
    \end{array}\)

    Hãy trả lời câu hỏi trước khi xem đáp án và lời giải

Câu hỏi này thuộc đề thi trắc nghiệm dưới đây, bấm vào Bắt đầu thi để làm toàn bài

ADSENSE/
QUẢNG CÁO
 

 

CÂU HỎI KHÁC

NONE
OFF